letsthinkcritically
letsthinkcritically
  • 327
  • 9 404 946

Відео

Equation on Sum of Powers
Переглядів 8 тис.Рік тому
Equation on Sum of Powers
Multiples of Primes | Irish National Mathematical Olympiad 2007
Переглядів 6 тис.Рік тому
Multiples of Primes | Irish National Mathematical Olympiad 2007
Solving This Equation With One Simple Trick
Переглядів 9 тис.Рік тому
Solving This Equation With One Simple Trick
Sum of Prime Squares is a Cube
Переглядів 7 тис.Рік тому
Sum of Prime Squares is a Cube
Solving This Problem With One Simple Trick
Переглядів 8 тис.Рік тому
Solving This Problem With One Simple Trick
When is This Fraction an Integer?
Переглядів 9 тис.Рік тому
When is This Fraction an Integer?
Equation of Powers Solved With One Simple Trick | Japan MO Finals
Переглядів 8 тис.Рік тому
Equation of Powers Solved With One Simple Trick | Japan MO Finals
Solving This Equation With One Simple Trick | Baltic Way 1992
Переглядів 6 тис.Рік тому
Solving This Equation With One Simple Trick | Baltic Way 1992
A Tricky Divisibility Problem
Переглядів 9 тис.Рік тому
A Tricky Divisibility Problem
Solving This Problem in One Step | Baltic Way 2011
Переглядів 34 тис.Рік тому
Solving This Problem in One Step | Baltic Way 2011
Equation on Symmetric Polynomials | Balkan MO 2017
Переглядів 9 тис.Рік тому
Equation on Symmetric Polynomials | Balkan MO 2017
When is p^2-p+1 a Cube? | Balkan MO 2005
Переглядів 13 тис.Рік тому
When is p^2-p 1 a Cube? | Balkan MO 2005
Are There Any Fifth Powers?
Переглядів 6 тис.Рік тому
Are There Any Fifth Powers?
An Interesting Equation of Powers
Переглядів 8 тис.Рік тому
An Interesting Equation of Powers
A Beautiful Equation | Switzerland IMO TST 2015
Переглядів 9 тис.Рік тому
A Beautiful Equation | Switzerland IMO TST 2015
A Quick System of Sum of Powers | Norwegian Abel Maths Competition
Переглядів 7 тис.Рік тому
A Quick System of Sum of Powers | Norwegian Abel Maths Competition
When is This Fraction a Cube?
Переглядів 11 тис.Рік тому
When is This Fraction a Cube?
A Nice Equation of Powers | Turkish National Maths Olympiad 2014
Переглядів 7 тис.Рік тому
A Nice Equation of Powers | Turkish National Maths Olympiad 2014
When the Sum of Powers are All Equal
Переглядів 8 тис.Рік тому
When the Sum of Powers are All Equal
2 Equations 3 Unknowns | Turkish Junior Mathematics Olympiad 2021
Переглядів 6 тис.Рік тому
2 Equations 3 Unknowns | Turkish Junior Mathematics Olympiad 2021
A Quick Functional Equation | India IMO TST 2010
Переглядів 10 тис.Рік тому
A Quick Functional Equation | India IMO TST 2010
Parabolas Solved in Two Ways
Переглядів 5 тис.Рік тому
Parabolas Solved in Two Ways
Indian National Mathematics Olympiad 2013 Problem 2
Переглядів 6 тис.Рік тому
Indian National Mathematics Olympiad 2013 Problem 2
A Quick Problem of Ratios
Переглядів 8 тис.Рік тому
A Quick Problem of Ratios
The Answer is Surprisingly Easy! | India National Mathematics Olympiad 2003
Переглядів 17 тис.Рік тому
The Answer is Surprisingly Easy! | India National Mathematics Olympiad 2003
One Simple Trick to Solve This National Maths Olympiad Problem
Переглядів 9 тис.Рік тому
One Simple Trick to Solve This National Maths Olympiad Problem
Could You Make The Greece IMO Team? | Greece IMO TST 2013
Переглядів 7 тис.Рік тому
Could You Make The Greece IMO Team? | Greece IMO TST 2013
When Can This be a Perfect Square? | Turkish National Mathematical Olympiad 2009
Переглядів 13 тис.Рік тому
When Can This be a Perfect Square? | Turkish National Mathematical Olympiad 2009
Turkish National Maths Olympiad 2004
Переглядів 7 тис.Рік тому
Turkish National Maths Olympiad 2004

КОМЕНТАРІ

  • @saswatipal2504
    @saswatipal2504 10 годин тому

    The more difficult version is the set of such sequence of 1's is never a perfect prime power

  • @xaxi222
    @xaxi222 День тому

    If n-m must be multiple of 100 (i.e. n-m≥100) and also m≥1, doesn't that mean that min(n+m) is for n=101 and m=1, thus min(n+m)=102? And obviously 101=n≥3, so that also checks out in this case, so isn't the answer 102 instead of 106 or am I missing something?

  • @user-tj1yr7gs5e
    @user-tj1yr7gs5e 4 дні тому

    Лемма Титу.

  • @lifeisacompetation
    @lifeisacompetation 4 дні тому

    Keep in mind "an Asian" Is solving this!

  • @xaxi222
    @xaxi222 4 дні тому

    Given the condition that y≥4 at 2:43 then the corresponding solution at 3:30 should indeed be (y,z)=(2t,3t), where for t∈ℕ, but only for t>1 (for t=1 => y=2<4). But one might then think "but (x,y,z)=(1,2,3) is obviously a solution". Yeah, but it's a result from the solution found at 6:46, not the one at 3:30

  • @BesnikDule-fl4yb
    @BesnikDule-fl4yb 4 дні тому

    I think was a very easy imo problem.

  • @xaxi222
    @xaxi222 5 днів тому

    The expansion of (a(x+1)+b)(cx+d) at 5:30 should have a bcx term instead of the bdx term shown in the video

  • @calculus-is-fun
    @calculus-is-fun 7 днів тому

    do i have any problem for this solution? since proven f is 1-1 and f(x+1987)=f(x)+1987, then f(x+1987)-f(x)=1987, hence f is a linear function (since the difference between every set of (x,x+1987) must be a constant), which x>0. Then, since f(f(x))=x+1987, m(mx+b)+b=x+1987, which we can get m=1,-1(rejected) and b=1987/2. considering that previously we know x>0 and f defined in Z>0, then looking back to the f(x)=x+1987/2, which firstly f is not integer for integer inputs, and it is also not defined in [0,1987/2). Therefore, we can claim that no such function.

  • @lechaiku
    @lechaiku 8 днів тому

    I've created 2 years ago a shortcut for such math problems: √[n (n+1) (n+2) (n+3) + 1] ------> for 4 consecutive numbers + 1 5n + (n -1 )^2 --------------------- 5 * 1000 + 1000000 - 2000 + 1 = 1000000 + 3001 = 1003001

  • @graham741
    @graham741 8 днів тому

    LHS must be even assuming it's not an even prime (but p=2 works too) so then RHS is odd, meaning it's congruent to 0 mod 4. Therefore p^3 + 1 = 0 mod 4 so p must be 3 mod 4, and primes must be 1 or 5 mod 6, so then p must be 11 or 7 mod 12. The only primes generated by this are 7,11,19,23,31, and 43 and checking yields p = 7,11 (it's annoying if you don't have a calculator but very doable with some mod checks + divisibility checks, eg if it's congruent to 0 modulo by n and not n^2 where n is not a perfect square then it cannot be a perfect square)

  • @mathsandillathensclasses634
    @mathsandillathensclasses634 9 днів тому

    I've the easiest and shortest method. It can be solved in a minute or two.

  • @Krish_202
    @Krish_202 12 днів тому

    How about using weirstrass inquealitiy considering a2a3......an=1 but its not given that a2a3........an<1

  • @niom9446
    @niom9446 17 днів тому

    bro is insane

  • @Sherif-Academy
    @Sherif-Academy 18 днів тому

    ua-cam.com/video/7-GkLcIZqiQ/v-deo.html outstanding!!!

  • @minhphamnhat7991
    @minhphamnhat7991 19 днів тому

    Where is 1/659?

  • @karolkurek9201
    @karolkurek9201 23 дні тому

    I didn't grab a finer point: how you went from 2^(x-1) congruent to 0 mod 8 to y+1+2^(x-1)=7*2^(2x-3). Can someone explain why k must be 2x-3, not 2x-1 or 2x+7 and so on?

  • @lechaiku
    @lechaiku 23 дні тому

    Another approach without using modulo and log (this is "no-pen-no-paper" method): 1. the last digit of x must be 9, because only 9 goes in cycles (9,1) with the 13th power being 9. 2. 100^13 is 27-digit number, so x must be 2-digit number ----> 26(digits) : 13 = 2. 3. x can't be 99, because the given 26-digit number is too small (it's obvious). 4. 8 = (2^3)^13 = 2^39; let's calculate 2^40 5. 2^10 = 1024, let's use "1000" 6. 2^40 = approx. 8^13 = approx 1000 ^4 7. 80^13 = approx 1000 ^4 * 10^13 = 10^12 * 10^13 = 10^25 8. It is less than the given 26-digit number 9. therefore x = 89

  • @drynshockgameplays
    @drynshockgameplays 24 дні тому

    How would you choose the squares if you got something like m^4 + 3m^3 + ... with the cubic term

  • @eloaba57
    @eloaba57 26 днів тому

    I got lost on the sumation part (note im finishing calc 1 so xd)

  • @MrEliseoD
    @MrEliseoD 27 днів тому

    Two cool facts about 13th powers is that a) the answer is approximately 13 times the length of the number; and b) that the last digit is always the last digit of the number we’re after… Given this, we can confidently say this is a two digit number ending with 9 (26 digits long, very convenient!), given 100^13 has 27 digits! The digit sum is not divisible by 3 or 9, so that leaves 39, 69 and 99 out… Another trick to know is that for 13th powers the second to last digit (which we can call x) can be used to determine the second to last digit of the root… in the case of a digit ending with 9, the value of the root’s second to last (technically first) digit would be 7(x-2) mod 10… equal to 7 x (6-2) mod 10 = 8… that would mean in our case that the 13th root of our number is 89…

  • @michaelg3490
    @michaelg3490 27 днів тому

    Can someone help me with where 2p<=q+6 comes from? Thank you!

  • @user-zf4si4go7t
    @user-zf4si4go7t 27 днів тому

    Vieta jumping and we done

  • @sproutssupport7789
    @sproutssupport7789 27 днів тому

    And why did you add u? When you can easly 1²-2×1×√2+1+(√2+1)²

  • @pichamonnaksomboon
    @pichamonnaksomboon 28 днів тому

    ทำไปแล้ว

  • @user-fd5ey5ch5c
    @user-fd5ey5ch5c 28 днів тому

    Hk?

  • @marcogallardo-my7hb
    @marcogallardo-my7hb 29 днів тому

    (3,4,5) and (3,5,6) and (4,3,5) and ( (4,5,3) and (5,3,4) and (5,4,3) is Solution. => n! Is Solution for n= 3, 3!=6 => For one Solution => 6 add. Total 30 solutions combined

  • @michaelaristidou2605
    @michaelaristidou2605 Місяць тому

    Couldn't A1 = B ? Because if it could, then we don't have a contradiction.

  • @Andrei-sw3my
    @Andrei-sw3my Місяць тому

    My solution : If you subtract 1 from a multiple of seven , that number will either be odd or factor into an odd * even . As 2^n is always even * even we can never have this relation .

    • @xinpingdonohoe3978
      @xinpingdonohoe3978 23 дні тому

      You'll have to prove that it might factor into odd×even. You don't have to prove it could be odd, because it will obviously either be odd or even, but you must show that this 7k-1 always has an odd factor.

    • @Andrei-sw3my
      @Andrei-sw3my 23 дні тому

      @@xinpingdonohoe3978 to prove just take it by cases : 1. K is even then k = 2m, we have 7(2m)-1 = 14m-1=odd. Case 2. K is odd then k= 2m + 1 , we have 7(2m+1)-1 = 14m - 6 = 2(7m-3) = even * odd . And as 2^n Is always even * even we cannot have this relation .

  • @Andrei-sw3my
    @Andrei-sw3my Місяць тому

    My solution : If you subtract 1 from a multiple of seven , that number will either be odd or factor into an odd * even . As 2^n is always even * even we can never have this relation .

  • @mickerson3979
    @mickerson3979 Місяць тому

    This solution is not good.

  • @MyOneFiftiethOfADollar
    @MyOneFiftiethOfADollar Місяць тому

    I just learned that we can think "critically" in 6 minutes. In school, critical thinking was often associated with "long thinks".

  • @mickerson3979
    @mickerson3979 Місяць тому

    It is not correct and your english is very bad. You did not solve this problem.

  • @MyOneFiftiethOfADollar
    @MyOneFiftiethOfADollar Місяць тому

    An ancillary result you "proved" here was that the order of an integer mod n divides. Phi(n). Nice job. People will consider subscribing to your channel if they like the content. No need to beg a the beginning and end for subscriptions.

  • @ToTheToTheToThe-pe6sz
    @ToTheToTheToThe-pe6sz Місяць тому

    Discriminant is so overpowered on math

  • @dhruvbhatia6808
    @dhruvbhatia6808 Місяць тому

    B cannot be equal to a due to them being integral Hence root b less than root a 2 root b less than root 2009 B less than 2009 / 4 Now u do the thing done in the video and u get a = 2009 + b - 2 root 2009×b Hence for (a) to be integer b = 41 k sq 41k sq less than 2009/4 Hence k = 0 1 2 3 So here we get all solutions in which a greater than b Same we do for b greater than a and get all the 8 solutions

  • @manalmachraa
    @manalmachraa Місяць тому

    pourquoi 1979 ne divise pas b ?

  • @mumtrz
    @mumtrz Місяць тому

    Here's a homework: a^b + b^c + c^a = (abc)^abc

    • @AyushKumar-ot3zr
      @AyushKumar-ot3zr Місяць тому

      do it urself

    • @mumtrz
      @mumtrz Місяць тому

      @@AyushKumar-ot3zr you think I'd give homework without knowing the answer myself? 🤣

    • @KaranSingh-qw7cn
      @KaranSingh-qw7cn 25 днів тому

      How to solve it ? Give me a hint

    • @amgalanbayarshagdar9435
      @amgalanbayarshagdar9435 20 днів тому

      This equation has only 3 solutions that [a; b; c]= [2; 1; 1], [1; 2; 1], [1; 1; 2]

    • @amgalanbayarshagdar9435
      @amgalanbayarshagdar9435 20 днів тому

      ​@@KaranSingh-qw7cn use equality that a=>b=>c

  • @ohiorizzler1434
    @ohiorizzler1434 Місяць тому

    wow! this is rizztastic!

  • @oof-inator282
    @oof-inator282 Місяць тому

    Does my solution work? (a+2b+3c+4d) is less than or equal to (a+b+c+d+a+b+c+d+2d)=(2+2d). Since 2d is at most 2*0.25, (a+2b+3c+4d) is less than or equal to 2+0.5=2.5. Now (a^a * b^b * c^c * d^d) is less than or equal to (abcd)^d since a smaller exponent would increase a number between 0 and 1. Also note that, (a^a * b^b * c^c * d^d) is less than or equal to (a)^(a+b+c+d)=a. Now, this is exactly equal when a=d or d=0.25. So now we can consider (abcd)^d with d=0.25 because that is when it is largest as per the previous inequality. By the AM-GM inequality, (abcd)^0.25 is less than or equal to (a+b+c+d)/4=0.25. Finally 2.5*0.25=0.625 so it's less than one.

  • @takyc7883
    @takyc7883 Місяць тому

    wow pairing the terms up is genius

  • @ItsJustEthan1
    @ItsJustEthan1 Місяць тому

    4:23 ish I think you have to consider the fact that the lim as t approaches 0 u approaches + OR MINUS infinity. Don't think this matters because that limit would still approach 0 after evaluating the other case so I think the result is still 0 but I think you need this to be rigorous.

  • @niom9446
    @niom9446 Місяць тому

    were you an IMO candidate?

  • @lupifa4395
    @lupifa4395 Місяць тому

    Since it's only integers, calculate f(f(n+1)) and get f(n+1)-f(n)... Who knows what you could possibly get... something recognisable, by any chance ?

  • @ElieBensaid
    @ElieBensaid Місяць тому

    14 mins just to study that number in 3 different mods, kind of a waste of time

  • @beautifulworld6163
    @beautifulworld6163 Місяць тому

    Wow thats so cool.

  • @ahmed-bechirbenhammouda370
    @ahmed-bechirbenhammouda370 Місяць тому

    I dont get this isnt there a function as in f(x)= x+993.5 ? f(f(x)) would be x+993.5+993.5=x+1987 ?? Please explain this to me and thanks

    • @low-litlight3438
      @low-litlight3438 Місяць тому

      This doesn’t work because the problem is about functions that map integers to integers. If you add a .5 anywhere then you map an integer to a non-integer, so it doesn’t work. Hope that helps.

  • @Kyojuro-bf3bf
    @Kyojuro-bf3bf Місяць тому

    Isnt it possible to first ignore the irrationality condition, then find all functions for all reals such that f(ab) = f(a+b). The only such function satisfying this is f(x) = c, a constant. So it also satisfies for any irrational a and b since the irrationals are a subset of the reals. ?

  • @rk-ds4vl
    @rk-ds4vl Місяць тому

    Very good

  • @sinox5
    @sinox5 Місяць тому

    "so yay we are done" - every math olympic when they finish a problem